Vous êtes sur la page 1sur 24

Question 1 : The boiling point of water is measured four times.

The
results are 110.01C, 110.02C, 109.99C and 110.01C. Which of the
following statements best describes this measuring process?
i. Accurate but not precise
ii. Precise but not accurate
iii.Neither accurate nor precise
iv.Both accurate and precise

Ans : Precise but not accurate.


Explanation: The measurements are close together, and thus
precise, but they are far from the true value of 100C, and thus
not accurate

(2) The length of an object is given as 3.21 0.02 cm. True or false:
Measurements = Measured value (uncertainty)
a. The length was measured to be 3.21 cm.
Ans: True
b. The true length of the object is 3.21 cm.
Ans: False
c. The bias in the measurement is 0.02 cm.
Ans: False
d. The uncertainty in the measurement is 0.02 cm.
Ans: True

(3) A person stands on a bathroom scale. The reading is 150 lb. After the
person gets off the scale, the reading is 2 lb.
a) Is it possible to estimate the uncertainty in this measurement? If so,
estimate it. If not, explain why not.
b) Is it possible to estimate the bias in this measurement? If so, estimate
it. If not, explain why not.
a) No, it is not possible to determine the standard deviation of the
process from a single measurement.
b) Yes, As the scale reads 2 pounds when the true weight is 0, the bias
can be estimated to be 2 pounds.

4: Assume that X and Y are independent measurements


with uncertainties X =0.2 and Y =0.4. Find the
uncertainties in the following quantities:
a. 3X
b. XY
c. 2X+3Y

3 X 3 X 3 0.2 0.6
X Y 0.2 0.4 0.447
2
X

2
Y

2 X 3Y 4 X2 9 Y2 4 0.2 2 9 0.4 2 1.2649

5) The length of a rod is to be measured by a process whose


uncertainty is 3 mm. Several independent measurements will be
taken, and the average of these measurements will be used to
estimate the length of the rod. How many measurements must be
made so that the uncertainty in the average will be 1 mm?

X = n
Sample mean uncertainty =

= 1 mm

Process measured uncertainty = = 3 mm

No of measurements = n =

( ) ( )

3
1

=9

(6) In the article The Worlds Longest Continued Series of Sea Level
Observations (M. Ekman, Paleogeography, 1988:7377), the mean annual
level of land uplift in Stockholm, Sweden, was estimated to be 4.93 0.23 mm
for the years 17741884 and to be 3.92 0.19 mm for the years 18851984.
Estimate the difference in the mean annual uplift between these two time
periods, and find the uncertainty in the estimate.

X Y X2 Y2 0.232 0.192 0.30 mm

7. A force of F = 2.2 0.1 N is applied to a block for a period of time,


during which the block
moves a distance d = 3 m, which is
measured with negligible uncertainty. The work W is given by W = F d.
Estimate W, and Find the uncertainty in the estimate?
Answers : Estimate of the workdone :- W = F d
F 3 0.1 N , d 3 m
W F d 2.1 3 6.3 Nm

W 3 F 3 0.1 0.3

Uncerta inty in estimate of workdone 6.3 0.3 Nm

If X is a measurement and
c is a constant, then

cX c X

8. Find the uncertainty in Y , given that X = 4.0 0.4 and


(a) Y = X2
(b) Y =X
(c) Y = 1/X
(d) Y = ln X
(e) Y = eX
(f ) Y = sin X (X is in units of radians)

(9) The volume of a cone is given by V = r2h/3, where r is the radius of


the base and h is the height. Assume the height is 6 cm, measured
with negligible uncertainty, and the radius is r = 5.00 0.02 cm.
Estimate the volume of the cone, and find the uncertainty in the
estimate.

V = dV
r
dr

Height = h = 6 cm
Radius = r = 5 cm

Measured uncertainty =

r = 0.02 cm

Volume of cone = V = r2h/3 = 652/3 = 157.0796 cm3


dV
dr = 2rh/3 = 265/3 = 62.832

dV
=
V dr r

V = 62.832 0.02 = 1.3 cm3

T 2

L
g

(10). The period T of a simple pendulum is given by


where L is the length of the pendulum and g is the acceleration due to
gravity.
a. Assume g = 9.80 m/s2 exactly, and that L 0.742 0.005 m . Estimate T ,
and find the uncertainty in the estimate.
b. Assume L = 0.742 m exactly, and that L 1.73 0.05 m . Estimate g, and
find the uncertainty in the estimate.
(a)

g 9.80, L 0.742, L 0.005, T 2

L
0.742
2
1.7289
g
9.80

dT
1 1

1.165204
dL
2 Lg
0.742 9.8
T

dT
L 1.165204 0.005 0.0058
dL

T 1.7289 0.0058 s

(b)

L
2 0.742
L 0.742, T 1.73, T 0.01, g 4 2 4
9.79
2
T
1.73
2

dg
2 L 8 2 0.742
2
4 3
11.315
3
dT
T
1.73
dg
g
T 11.315 0.01 0.11
dT

g 9.79 0.11 m/s 2

T 1.7289 0.0058 s

c. Assume g = 9.80 m/s2 exactly, and that L 0.855 0.005 m . Estimate


T , and find the relative uncertainty in the estimate.
d. Assume L = 0.855 m exactly, and that T 1.586 .11 s . Estimate g, and
find the relative uncertainty in the estimate.
(c)

g 9.80, LL =0.855
0.742, L 0.005, T 2

L
0.855
1.8588
2
1.588558
g
9.80

dT
1
1

1.0853
dL
2
Lg
0.855 9.80

dT
L 1.0853 0.005 0.005426
dL

Relative uncertainty in T

T
T

0.005426
0.002923
1.588558
1.8588

L 0.855 T 1.856 0.005


L
0.855
T 2
2
1.856 g 9.7987
g
g

T2
g
;
2
4 L

dg
2T

dT 4 2 L

dg
2T
2 1.856
g
T 2 T 2
0.005 5.4986 10 4
dT
4 L
4 0.855

5.4986 104
100
100 0.00561%
g
9.7987

11. Convert the following absolute uncertainties to relative

uncertainties.
a. 37.2 0.1
b. 8.040 0.003

c. 936 37
d. 54.8 0.3
Ans:

a) The relative uncertainty = 0.1/37.2 = 0.27%


b) The relative uncertainty = 0.003/8.040 = 0.0373%
c) The relative uncertainty = 37/936 =3.953%
d) The relative uncertainty = 0.3/54.8 = 0.547%

(12) The acceleration g due to gravity is estimated by dropping an


object and measuring the time it takes to travel a certain distance.
Assume the distance s is known to be exactly 2.2 m. The time is
measured to be t = 0.67 0.02 s. Estimate g, and find the relative
uncertainty in the estimate. (Note that g = 2s/t2)
Time = t = 0.67 sec

Distance = s = 2.2 m

Measured uncertainty =

Y = dY
dX X

t = 0.02 sec

g = 2s/t2 = 22.2/0.672 = 9.802 m/sec2


dg -22s
-222.2
= -29.259
=
=
dt
t3
0.673

dg
g = dt t

g = -29.259 0.02 = 0.585 m/sec2

0.585
% g = g = 9.802 = 0.06 = 6.0 %

g = 9.802 m/sec2 6.0 %

13

14 From a fixed point on the ground, the distance to a certain tree is


measured to be s = 55.2 0.1 m and the angle from the point to the
top of the tree is measured to be = 0.50 0.02 radians. The height of
the tree is given by h = s tan .
a. Estimate h, and find the uncertainty in the estimate.
b. Which would provide a greater reduction in the uncertainty in h:
reducing the uncertainty in s to 0.05 m or reducing the uncertainty
in to 0.01 radians?
Angle = = 0.5 radian

= 0.02 radian

Distance = s = 55.2 m
h = s tan

s = 0.1 m

a. Estimate h, and find the uncertainty in the estimate.

h = s tan = 55.2 tan(0.5) = 55.2 0.546 = 30.156 m


h = s sec2 = 55.2 sec2(0.5) = 55.2 1.1392 =71.67

h = tan = tan(0.5) = 0.546


s

h = (0.5462 0.12 +71.672 0.022)0.5 = (0.003+2.055)0.5 = 1.434


h = 30.156 1.434 m

b. Which would provide a greater reduction in the uncertainty in h:


reducing the uncertainty in s to 0.05 m or reducing the uncertainty
in to 0.01 radians?

s = 0.05 m
= 0.02 radian

s = 0.1 m
= 0.01 radian

h =71.67

h = 0.546
s

h =71.67

h = 0.546
s

h = (0.5462 0.052 +71.672 0.022)0.5


= (0.0007+2.055)0.5 = 1.434

h = (0.5462 0.12 +71.672 0.012)0.5


= (0.003+0.514)0.5 = 0.719

Reducing the uncertainty in to 0.01 radians provides the greater reduction.

(15) When air enters a compressor at pressure P1 and leaves at pressure


P2, the intermediate pressure is given by P3 P1P2 . Assume that
P1 10.1 0.3MPa and P2 20.1 0.4MPa

a. Estimate P3,andfindthe uncertainty in the estimate.


b. Which would provide a greater reduction in the uncertainty in P3:
reducing the uncertainty in P1 to 0.2 MPa or reducing the
uncertainty in P2 to 0.2 MPa?
(a)

P1 10.1, P1 0.3, P2 20.1, P1 0.3, P3 PP


1 2 14.25
P3
0.5 P2 / P1 0.705354
P1
P3
0.5 P1 / P2 0.354432
P2
2

P3 2 P3 2
P1
P 2 0.25
3
P1
P2
P3 14.25 0.25MPa

(b)

P1 10.1, P2 20.1, P3 PP
1 2 14.25
P3
0.5 P2 / P1 0.705354
P1
P3
0.5 P1 / P2 0.354432
P2
2

P3 2 P3 2
P3
P1
P 2 0.25
P1
P2
When P1 0.2 and P2 0.4, then

P
3

0.705354

0.22 0.3544322 0.42 0.20

When P1 0.3 and P2 0.2, then

P
3

0.705354

0.32 0.3544322 0.22 0.22

Reducing the uncertainty in P1 to 0.2 MPa provides the greater reduction.

16. The lens equation says that if an object is placed at a distance p from a lens, and an image
is
formed at a distance q from the lens, then the focal length f satises the equation
1/f = 1/p + 1/q. Assume that p = 2.3 0.2 cm and q = 3.1 0.2 cm.
a. Estimate f and nd the uncertainty in the estimate.
b. Which would provide a greater reduction in the uncertainty in f : reducing the
uncertainty in p to 0.1 cm or reducing the uncertainty in q to 0.1 cm?

Answers : a.) Estimate f and its uncertainty :

1 1 1
1
1

0.7573
f p q 2.3 3.1
pq
f
pq

f 1.320

U 2
U 2 U 2
X n
X 1
X 2 .......
U

X
1
2
n

f
f
f 2p q2
p
q
2

1 1 1

f p q

q 2 2 p2 2

2.32

3.12
2
2

0
.
2

0
.
2
p
q
2
2.3 3.12
2.3 3.12
p q 2

p q

f 4.4344e 3 1.3164e 3 0.0758

f 1.320 0.0758

b. reduction in the uncertainty in f : reducing the uncertainty in p to 0.1 cm or reducing


the uncertainty in q to 0.1 cm
If p = +/- 0.1 and q = +/- 0.2
2

q 2 2 p2 2

2.32

3.12
2
2

0
.
1

0
.
2
p
2
2.3 3.12
2.3 3.12
p q 2 q

p q

f 1.086e 3 1.3164e 3 0.04901

f 1.320 0.04901
If p = +/- 0.2 and q = +/- 0.1
2

q 2 2 p2 2

2.32

3.12
2
2

0
.
2

0
.
1
p
q
2
2.3 3.12
2.3 3.12
p q 2

p q

f 4.4344e 3 3.2910e 3 0.06901

f 1.320 0.06901
Reduction in p uncertainty will increase accuracy

Vous aimerez peut-être aussi